Investment Review Questions COPY Flashcards

1
Q
Which of the following is not covered by the indenture agreement?
A. Bond quality
B. Amount of issue
C. Property pledge (if any)
D. Call provisions
A

A - The rating agencies rate the bonds for quality.

How well did you know this?
1
Not at all
2
3
4
5
Perfectly
2
Q

Which of the following $1,000 par bonds is selling at a premium?
A. $952
B. $1052

A

B - Purchase price is in excess of par value.

How well did you know this?
1
Not at all
2
3
4
5
Perfectly
3
Q

If a bond is selling at a premium, which of the following is true?
A. Its yield to call exceeds the current yield.
B. Its current yield exceeds the yield to maturity.
C. Its current yield has risen.
D. The bond cannot be called.

A

B - The answer is from the yield ladder.

How well did you know this?
1
Not at all
2
3
4
5
Perfectly
4
Q
Which of the following is not an investment grade bond?
A. BBB
B. BB
C. AAA
D. Aa
A

B

How well did you know this?
1
Not at all
2
3
4
5
Perfectly
5
Q
Which of the following is a bond rating company?
A. GNMA
B. S&P
C. A.M. Best
D. BRC
E. FNMA
A

B - Standard & Poor’s (S&P) and Moody’s are bond rating companies.

How well did you know this?
1
Not at all
2
3
4
5
Perfectly
6
Q
Cindy owns a bond with a par value of $1,000. The bond matures in 5 years. The bond has a 7% coupon (paid semiannually). Comparable debt yields 8%. What is the intrinsic value of Cindy's bond?
A. $959.45
B. $970.80
C. $1,000.00
D. $1,041.58
A
A - HP 10B
HP 12C
HP 17BII
$1,000, FV
$1,000, FV
$1,000, FV
$70, Ö, 2, PMT
$70, enter, 2, Ö, PMT
$70, Ö, 2, PMT
5, gold, xP/YR
5, enter, 2, x, n
5, gold, N
8, I/YR
8, enter, 2, Ö, i
8, I%YR
PV
PV
PV
How well did you know this?
1
Not at all
2
3
4
5
Perfectly
7
Q

Which of the following is true about EE bonds?
I. They are marketable investments.
II. They are purchased at their face value.
III. Investors can declare the interest annually or at redemption.
IV. Interest is subject to federal income tax.
V. Interest is paid semiannually.
A. I, III, IV, V
B. I, III, IV
C. II, III, IV
D. II, V

A

C - EEs are not marketable investments. They are now purchased at full face value.

How well did you know this?
1
Not at all
2
3
4
5
Perfectly
8
Q
Which of the following securities is not a direct guarantee of the U.S. government?
A. STRIPS
B. GNMA
C. CMO
D. EE bond
A

C - The other 3 investments are guaranteed.

How well did you know this?
1
Not at all
2
3
4
5
Perfectly
9
Q

Which of the following is true about GNMAs?
I. If mortgage rates decrease, prepayment may increase.
II. The amount received each month can vary.
III. The certificates are guaranteed by the U.S. Government.
IV. Payments include interest and principal.
V. The realized yield on the bonds can be somewhat variable.
A. All of the above.
B. II, III, IV
C. II, IV
D. III, IV, V
E. I, III, IV

A

A - The amount received each month and the realized yield on the certificates are somewhat variable
because of principal prepayments.

How well did you know this?
1
Not at all
2
3
4
5
Perfectly
10
Q

Mrs. Lanier (35% federal tax bracket) lives in New York City. She bought EE education bonds for her grandchildren (who also live in New York City) some years ago. If she redeems the bonds for her grandchildren’s education, taxation will be which of the following?
A. Interest taxable at federal, state and local rates (her bracket)
B. Interest taxable at federal, state, and local rates (her grandchildren’s bracket)
C. Interest taxable at federal rates (her bracket)
D. Interest taxable at state and local rates (her bracket)
E. No tax

A

C - In order to qualify for the education interest exclusion, the taxpayer generally must be the parent. The grandparent can take the exclusion only if the grandchild is the grandparent’s dependent (can’t make that assumption). At a 35% federal tax bracket, her income exceeds the EE phaseout. The bond’s interest is subject to federal but not state and local taxes.

How well did you know this?
1
Not at all
2
3
4
5
Perfectly
11
Q
Which instrument is used to finance imports and exports?
A. Euro dollars
B. ADRs
C. Yankee bond
D. Banker's acceptance
A

D Banker’s Acceptance.

How well did you know this?
1
Not at all
2
3
4
5
Perfectly
12
Q

Which of the following is true about I bonds?
I. Series I bonds earn interest up to in 30 years.
II. Series I bonds accrue earnings based on both a fixed rate of return and the semiannual inflation rate.
III. The special tax benefits available for education savings with Series EE bonds also apply to Series I bonds.
IV. The difference between the purchase price and the redemption value is taxable interest (redeemed or matures).
A. All of the above
B. I, II
C. I, II, III
D. II, IV
E. III, IV

A

A - I bond earnings are based on both a fixed rate of return and the semiannual inflation rate. I bonds are taxed the same way EEs are taxed. Like EE bonds, the I bonds only earn interest for up to 30 years.

How well did you know this?
1
Not at all
2
3
4
5
Perfectly
13
Q

Mrs. Smith has the following CDs at one bank. How much is covered by FDIC insurance?
I. $50,000 in joint tenancy with her daughter
II. $50,000 in joint tenancy with her son
III. $50,000 in joint name with her husband
IV. $100,000 in her name
A. $175,000 B. $200,000 C. $225,000 D. $250,000

A
D - Jointly
Mrs. Smith
Other
$25,000
$25,000
daughter
$25,000
$25,000
son
$25,000
$25,000
husband
$ 75,000
$75,000
Mrs. Smith's jointly held accounts don't exceed $100,000. Her jointly held account ($75,000) plus other jointly held accounts ($75,000) plus her separate account ($100,000) are covered by FDIC insurance.
How well did you know this?
1
Not at all
2
3
4
5
Perfectly
14
Q
Mrs. Patrick has the following assets at one FDIC?insured bank.
Asset
Various Certificates of Deposit
Ownership
Mrs. Patrick
Balance
$100,000
Money Market Deposit Account
Mrs. Patrick
$50,000
IRA Rollover
Mrs. Patrick
$200,000
Passbook Savings
Joint with son
$100,000
Checking Account
Joint with daughter
$100,000
Savings Account
Joint with husband
$250,000
How much is currently insured by the FDIC?
A. $725,000
B. $750,000
C. $800,000
D. $825,000
A

C - She is insured for $150,000 for the CD and money market and $200,000 for the IRA. The joint accounts are handled as follows.
Mrs. Patrick Others
Joint with son $50,000 $50,000
Joint with daughter $50,000 $50,000
Joint with husband $125,000 $125,000
$150,000 single + $200,000 RA + $225,000 JT + $225,000 JT = $800,000
Remember: Insurance coverage is per titling, not per account.

How well did you know this?
1
Not at all
2
3
4
5
Perfectly
15
Q
Mrs. Able has the following assets at one FDIC?insured bank.
Asset
Ownership
Balance
Various Certificates of Deposit
Mrs. Able
$300,000
Money Market Deposit Account
Mrs. Able
$50,000
IRA Rollover
Mrs. Able
$200,000
Passbook Savings
Joint with son
$100,000
Checking Account
Joint with daughter
$100,000
Savings Account
Joint with husband
$400,000
How much is currently insured?
A. $800,000
B. $950,000
C. $975,000
D. $1,000,000
E. $1,150,000
A

D - Mrs. Patrick Others
Joint with son $50,000 $50,000
Joint with daughter $50,000 $50,000
Joint with husband* $150,000 $200,000
$250,000 single + $200,000 I RA $250,000 $300,000
+ JT + JT $1,000,000
*With the $150,000 she has the maximum of $250,000 in joint accounts. Her husband gets 1/2 of
$400,000. The remaining $50,000 is not insured. Remember FDIC insurance is per titling, not per account.

How well did you know this?
1
Not at all
2
3
4
5
Perfectly
16
Q
Which of the following bonds produces the most income per initial cost using an annual coupon?
A. 6.5% coupon purchased for $800
B. 7.5% coupon purchased for $850
C. 10% coupon purchased for $1,100
D. 11% coupon purchased for $1,200
Investment Planning Quiz ? Lesson 1
A

D - A. $65 / $800 = 8.13% B. $75 / $850 = 8.82%

C. $100 / $1,100 = 9.09% D. $110 / $1,200 = 9. 17%

How well did you know this?
1
Not at all
2
3
4
5
Perfectly
17
Q

Sam purchased a unit trust. He is confused about some issues. Which of the following are true?
I. This is considered a passive investment technique.
II. The unit trust will self?liquidate.
III. Sam can trade the unit trust on the secondary market.
IV. Payments can be income and/or return of principal.
V. There is continuous offering and redemption.
A. All of the above
B. I, II, III, IV
C. I, II, IV
D. II, III
E. II, IV

A

B - There is not a continuous offering and redemption. That’s the wording for mutual funds. The UITs may be sold back to the sponsor.

How well did you know this?
1
Not at all
2
3
4
5
Perfectly
18
Q

Exchange traded fund is best described by which of the following?
A. Closed End Fund
B. Open End Fund
C. UIT
D. No load Balanced Mutual Fund
E. Mostly an open end fund but could be a closed end fund

A

E - The answer could be either A or B. Answer E, therefore, is the best answer. An ETF may be an open?end or closed? end fund. ETFs are traded on a major exchange. It is possible to find a secondary market for UITs units among brokers and dealers.

How well did you know this?
1
Not at all
2
3
4
5
Perfectly
19
Q
In which situation is the supply of shares limited?
A. Closed?End Fund
B. Open?End Fund
C. UIT
D. No?load Balanced Mutual Fund
E. B and D
A

A - UITs issue units, not shares.

How well did you know this?
1
Not at all
2
3
4
5
Perfectly
20
Q
What can always be purchased at NAV?
A. Closed?End Fund
B. Open?End Fund
C. UIT
D. No?load Balanced Mutual Fund
E. B and D
A

D - Answer D is the best answer since open?end funds could be load or no?load. Also, the word alwaysmakes Answer D more correct.

How well did you know this?
1
Not at all
2
3
4
5
Perfectly
21
Q
Once created, no new securities are purchased, and portfolio securities are rarely sold.
A. Closed?End Fund
B. Open?End Fund
C. UIT
D. No?load Balanced Mutual Fund
E. B and D
A

C

How well did you know this?
1
Not at all
2
3
4
5
Perfectly
22
Q
Shares are purchased and redeemed directly with the issuer.
A. Closed?End Fund
B. Open?End Fund
C. UIT
D. No?load Balanced Mutual Fund
E. B and D
A

E

How well did you know this?
1
Not at all
2
3
4
5
Perfectly
23
Q
Harry is interested in purchasing just one fund that will give him the highest diversification possible. Which fund do you suggest?
A. Global Fund
B. International Fund
C. 500 Index Fund
D. Growth Fund
A

A - Global funds would give Harry the greatest diversification (worldwide plus U.S. issues).

How well did you know this?
1
Not at all
2
3
4
5
Perfectly
24
Q
Your client is interested in purchasing of an apartment complex with the following anticipated financial characteristics:
?
Potential Gross Income (PGI)
= $2,500,000
?
Vacancy Rate
= 8% of PGI
?
Operating Expenses
= 28% of PGI
?
Capitalization Rate
= 12%
Based on this information, what is the maximum price you would advise the client to pay?
A. $ 1,600,000
B. $ 6,944,444
C. $13,333,333
D. $20,833,333
A

C - Potential Gross Income $2,500,000 Less vacancy (8%) and operating expenses (28%) of PGI ?900,000 (36%) Net Operating Income {NOI) $1,600,000
Intrinsic Value = NOI = $1.600.000 = 13,333,333.33 Cap Rate .12

How well did you know this?
1
Not at all
2
3
4
5
Perfectly
25
Q
What investment is most suitable for a client seeking growth and tax efficiency?
A. Limited Partnership
B. Municipal Bond
C. Government Bond
D. S&P Index Fund
A

D - Index funds seek to duplicate the S&P 500 Index. Turnover of stock is minimal; therefore, they are tax? efficient.

How well did you know this?
1
Not at all
2
3
4
5
Perfectly
26
Q

A global fund invests in which of the following?
A. Only non?U.S. investments
B. Both U.S. and foreign investments
C. Only corporations specializing in exported goods
D. Only corporations specializing in imported goods

A

B - Global funds invest in foreign and domestic securities. International funds invest in non?U.S. securities.

How well did you know this?
1
Not at all
2
3
4
5
Perfectly
27
Q
Which of the following corporate reports are filed with the SEC?
I. Corporate Annual Report
II. 10Q
III. 10K
IV. IRS Form 1120
A. I, II, III, IV
B. I, II, III
C. II, III
D. I
A

C - The Annual Report is sent to stockholders. Form 1120 is the corporate federal tax return.

How well did you know this?
1
Not at all
2
3
4
5
Perfectly
28
Q

An ADR is which of the following?
A. An instrument used to affect payment in import?export transactions
B. A receipt for shares of a foreign?based corporation
C. An instrument that contracts in the futures market for a foreign currency
D. A corporation organized under the laws of a foreign country

A

B

How well did you know this?
1
Not at all
2
3
4
5
Perfectly
29
Q

Your client currently owns stocks in various U.S. companies (25?30). The client expresses a desire to diversify his portfolio. Which of the following choices best achieves the greatest diversification and risk reduction by buying more investments?
A. Purchase additional bonds from the same companies but with different maturity dates (laddering)
B. Buy stocks in 5 more blue chip U.S. companies
C. Buy a Global Fund
D. Buy an International Fund

A

D - By buying the International, fund, the client improves diversification (low correlation). The Global Fund has some stocks. Bonds may have a low correlation, but they have risk (duration lesson). Come back to this question after reviewing duration (use a duration of 10 with a YTM of 4% and an interest change of
+1. Somewhat subjective.

How well did you know this?
1
Not at all
2
3
4
5
Perfectly
30
Q
A 13?unit apartment project costs $700,000. It has 13 two?bedroom apartments renting for $750 per month. Laundry income is $1,000 per year. Vacancy and collection losses are 7% of potential gross income. Operating expenses are $44,250 for this year. Calculate the yearly net operating income (NOI).
A. ($19,592)
B. $43,672
C. $64,560
D. $65,490
E. $65,560
A

D - 13 two?bedroom renting at $750/month = $9,750 x .12
Gross Rental Income = $117,000
Other Income = +1,000
Potential Gross Income (PGI) = $118,000
less vacancy and collection (7% PGI) = 8,260
Effective Gross Income = $109,740
less operating expenses* (total) = ? 44,250
Net Operating Income (NOI) = $ 65,490
*Depreciation and debt service are not used to compute NOI.

How well did you know this?
1
Not at all
2
3
4
5
Perfectly
31
Q
Using question #14 with a 12% cap rate, what is the maximum price you would advise the client to pay?
A. $545,750
B. $914,500
C. $975,000
D. $983,333
A

A - Intrinsic Value = NOI = $65,490 = 545,750 Cap Rate .12

How well did you know this?
1
Not at all
2
3
4
5
Perfectly
32
Q

In comparing index mutual funds to exchange traded funds (ETFs), ETFs have which of the following advantages?
I. ETFs can be bought on margin.
II. ETFs can be sold short.
III. ETFs can be bought or sold throughout the trading day.
IV. Trading orders can include stop?loss and limit orders.
A. All of the above
B. I, II
C. II, III, IV
D. III, IV
E. III

A

A - All of these are advantages unless you make a wrong decision and lose money.

How well did you know this?
1
Not at all
2
3
4
5
Perfectly
33
Q

How should your 35?year?old single client invest?
A. 35% aggressive growth fund, 50% international fund, 15% gold fund
B. 50% tech fund, 50% health fund
C. 100% growth and income fund
D. 35% S&P 500 fund, 35% growth fund, 30% high yield bond fund
Investment Planning Quiz ? Lesson 2

A

D - Answer D is arguably the best choice since we don’t know the client’s risk tolerance or tax bracket. Subjective.
Investment Planning Quiz ? Lesson 3

How well did you know this?
1
Not at all
2
3
4
5
Perfectly
34
Q
What type of option offers the highest potential for profit?
A. Writing a covered put
B. Writing a covered call
C. Writing a naked put
D. Writing a naked call
E. Buying a call
A

E - The upside is unlimited. Answer A is not a bad answer because the stock’s upside is unlimited, but to calculate the net profit of these positions (the short put and long stock), we must reduce the amount of gain by the original cost of the stock less the put premium income (see example below). For Answer E, the only offsetting cost is the call’s premium. The call premium is almost certainly lower than the cost of the stock (assuming the stock and call were purchased at the same time). Answer E is the best answer.
EXAMPLE: If I buy a naked call (100 shares) on a stock for $1,000 when the stock is $400 per share and it goes to $500, I make $100 x 100 shares = $10,000 for a $1,000 investment. In Answer A, if I bought the shares $40,000 sold for $50,000 and the option ($1,000), then I would have made $9,000. Cost is
$40,000 (shares) + $1,000 (option).

How well did you know this?
1
Not at all
2
3
4
5
Perfectly
35
Q
Which of the following strategies are profitable in a rising market?
I. Buying a call
II. Buying a put
III. Selling a naked call
IV. Selling a put
A. I, II
B. I, IV
C. II, III
D. III, IV
E. I
A

B - Buying a call and selling a put are profitable strategies in a rising market. Buying a put and selling a naked call are profitable in a falling market.

How well did you know this?
1
Not at all
2
3
4
5
Perfectly
36
Q

ABC owns a large citrus grove. ABC is concerned about an oversupply of South American juice being imported into the U.S. Which of the following hedge positions should ABC take?
A. A short hedge ? ABC should sell OJ futures contracts because it is hedging against lower OJ prices
B. A short hedge ? ABC should sell OJ futures contracts because it is hedging against higher OJ prices
C. A short hedge ? ABC should buy OJ futures contracts because it is hedging against lower OJ prices
D. A long hedge ? ABC should buy OJ futures contracts because it is hedging against lower OJ prices

A

A - When ABC owns the OJ, it is long in OJ (However, it does not yet have the orange juice. It will be long when the oranges are harvested and juice squeezed). ABC would hedge against lower orange juice prices in the future by going short, (selling) a contract of orange juice at today’s prices for delivery tomorrow (when selling prices may be lower). This would hedge against the South American juice reducing OJ prices.

How well did you know this?
1
Not at all
2
3
4
5
Perfectly
37
Q

Assume that Harry can choose between buying 100 shares of stock at $300 per share or buying a call option ($300 exercise price) for $20. How much can he lose if the stock falls by 25% and the option expires worthless?
I. If Harry buys and sells the stock, he will lose $7,500.
II. If Harry buys the option and it expires worthless, he will lose $2,000.
A. Both I and II
B. I
C. II
D. Neither I nor II

A

A - If the option expires worthless, the option is considered sold (at expiration) and is treated as a short? term loss. $30,000 x .25 = $7,500. The concept is even more important. He will lose less money with the option purchase (maximum 300). If he buys the stock, the loss is $7,500.

How well did you know this?
1
Not at all
2
3
4
5
Perfectly
38
Q

Which of the following statements about calls and puts are true?
A. A writer of a call expects the price to stay steady or perhaps rise in the near future.
B. When the market price of a stock is less than the exercise price of the option, the put is in the money.
C. The buyer of a naked call has unlimited loss potential.
D. If the underlying stock is trading at $55, the intrinsic value of a call with a strike price of $60 is
$5.

A

B - Answer A would have been correct if it said fall in the near future. Answer C is wrong. The seller of a naked call has unlimited loss potential. Answer D is wrong. The option is trading out?of?the?money (Intrinsic value is zero).

How well did you know this?
1
Not at all
2
3
4
5
Perfectly
39
Q
A put option with a strike price of $110 is selling for $3?1/2 when the market price of the underlying stock is $108. What is the intrinsic value of the put?
A.   0
B.   1?1/2
C.   2 D.   3?1/2
   E.   ?2
A

C - It is $2 in the money.

How well did you know this?
1
Not at all
2
3
4
5
Perfectly
40
Q

John is considering adding to his coin and stamp collection. Which of the following is true?
A. This is an efficient market.
B. Stamps and coins are more marketable than art and antiques.
C. Stamps and coins have a small bid?ask margin.
D. John would not be subject to the same elements of risk attributable to the stock market.
E. The return on physical assets is normally negatively correlated with returns on financial assets.

A

E - Inflation, while bearish to stocks and bonds, may be beneficial for collectibles. Answer D is a good answer, but Answer E is the best answer.

How well did you know this?
1
Not at all
2
3
4
5
Perfectly
41
Q

A client buys two puts.
October ABC put @$30 January XYZ put @ $40 ABC selling @ $32 XYZ selling @ $36 What is the intrinsic value of the two options?
A. $?2/$4 B. $0/$4 C. $?2/$0 D. $0/$?4

A

B - Think simple ? You cannot have a negative intrinsic value. Only one answer has no negative.

How well did you know this?
1
Not at all
2
3
4
5
Perfectly
42
Q

On July 30th, an XYZ DEC 55 call has a premium of $6?1/2, and XYZ shares have a market price of $58. Which of the following are true?
I. The call has no intrinsic value.
II. The call has an intrinsic value of $3.
III. The call has an intrinsic value of $3?1/2.
IV. The call has a time value of $3.
V. The call has a time value of $3?1/2.
A. I, V
B. II, V
C. III, IV

A

B - IV = MP (58) ? EP (55), 3 points in the money; therefore, it has an intrinsic value of $3. The rest of the premium represents time premium.

How well did you know this?
1
Not at all
2
3
4
5
Perfectly
43
Q
Bob bought a call option for $500. The option time period has expired. What is the market value of the option?
A. Zero
B. $1
C.   $500
D.   No solution
Investment Planning Quiz ? Lesson 3
A

A - It has expired. It is worthless.

Investment Planning Quiz ? Lesson 4

How well did you know this?
1
Not at all
2
3
4
5
Perfectly
44
Q
Which of the following is not a source of systematic risk?
A. Purchasing power risk
B. Liquidity risk
C. Interest rate risk
D. Market risk
E. Exchange rate risk
A

B - The answer is PRIME (answers A, C, D & E).

How well did you know this?
1
Not at all
2
3
4
5
Perfectly
45
Q
If you wanted to purchase a mutual fund that would have the lowest correlation with a U.S. common stock fund, which fund would you select?
A. Global fund
B. International fund
C. Emerging markets fund
D. Japan fund
E. European fund
A

C - The correlation coefficient between the U.S. market and an emerging markets fund is low.
Emerging markets: Less developed countries
Global: World (including US)
International: Non?U.S./foreign only

How well did you know this?
1
Not at all
2
3
4
5
Perfectly
46
Q
The risk level quantification of beta is which of the following?
I. Volatility
II. Systematic risk
III. Non?systematic risk
IV. Unsystematic risk
V. Total risk
A. I, II, V
B. I, II
C. II, V
D. III, IV
A

B - III, IV, V refer to standard deviation.

How well did you know this?
1
Not at all
2
3
4
5
Perfectly
47
Q
The standard deviation of variable X is .20. The standard deviation of variable Y is 0.12. The covariance between X and Y is 0.0096. This correlation between X and Y is which of the following?
A. .20
B. .24
C. .36
D. .40
A

D - .0096 Ö (.20 x .12) = .40

How well did you know this?
1
Not at all
2
3
4
5
Perfectly
48
Q
Which of the following investments (rate of returns shown) has the highest standard deviation?
Year
Stock #1
Stock #2
Stock #3
1
?10%
10%
0%
2
20%
15%
20%
3
30%
20%
20%
4
?15%
?10%
0%
A. Stock #1
B. Stock #2
C. Stock #3
A
A - Stock #1:
22.13
Stock #2:
13.15
Stock #3:
11.55
How well did you know this?
1
Not at all
2
3
4
5
Perfectly
49
Q
Stock ABC has an average (mean) return of 16% with a standard deviation of 16%. Within what range could an investor expect a return to fall 68% of the time?
A.   ?16% to 32% B.   0% to 32%
C.   16% to 32%
D.   0% to 16%
E.   32%
A

B - 16% ± 16% 0% to 32%

How well did you know this?
1
Not at all
2
3
4
5
Perfectly
50
Q

Which of the following investments is least risky?
Stock X
20% average return
Standard Deviation 3
Stock Y
25% average return
Standard Deviation 5
Stock Z
15% average return
Standard Deviation 2
A. Stock X because it has a lower coefficient of variation.
B. Stock Y because it has a higher coefficient of variation.
C. Stock Z because it has a lower coefficient of variation.
D. Stock X because it has a higher coefficient of variation.

A

C - CV = SD Ö mean (average return) X. 3% Ö 20% = .15
Y. 5% Ö 25% = .20
Z. 2% Ö 15% = .13
Z has the lowest coefficient of variation.

How well did you know this?
1
Not at all
2
3
4
5
Perfectly
51
Q

When are two investments perfectly positively correlated?
A. When the covariance is +1.0
B. When the correlation coefficient is 0.0
C. When the covariance is ?1.0
D. When the correlation coefficient is +1.0

A

D - The question is asking about the correlation.

How well did you know this?
1
Not at all
2
3
4
5
Perfectly
52
Q
The weighted beta of the following portfolio is which of the following?
Weighting
Beta
Stock 1
25%
1.2
Stock 2
35%
.5
Stock 3
40%
.9
A. .815
B. .835
C. .845
D. .85
A

B - 25% x 1.2 = .300
35% x .5 = .175
40% x .9 = .360
.835

How well did you know this?
1
Not at all
2
3
4
5
Perfectly
53
Q

Which of the following is true?
I. A negative correlation coefficient will reduce the portfolio risk.
II. A negative correlation coefficient will increase the portfolio risk.
III. A negative correlation coefficient will make the beta negative.
IV. A negative correlation coefficient will increase the portfolio standard deviation.
A. I, III
B. II, III
C. II, IV
D. III, IV
E. I

A

A - A negative correlation will reduce the portfolio beta. The formula for beta includes the correlation coefficient. When it is negative, the beta will be negative.

How well did you know this?
1
Not at all
2
3
4
5
Perfectly
54
Q
Tim owns two stocks, Companies A and B. Company A has just completed an acquisition of Company C. The standard deviation of stock A was unchanged by the acquisition. How will the covariance of two stocks (A and B) be affected if the new Company C is negatively correlated to Companies A and B?
A. Unchanged
B. Increased
C. Decreased
D. Becomes positive
E. Becomes negative
A

C - The covariance has a direct relationship to the correlation coefficient. The new acquisition ls negatively correlated with both Company A and B. That should reduce the covariance but not necessarily make it negative. It depends on the weighting of company A and B. A is weighted at 40% and B is weighted at 40%. They are highly correlated. So C is negatively correlated but it is only 20%. The weighting is still positive. The standard deviation formula uses weighting.

How well did you know this?
1
Not at all
2
3
4
5
Perfectly
55
Q
U.S. Treasury securities are subject to which of the following risks?
I. Credit Risk
II. Purchasing Power Risk
III. Marketability Risk
IV. Default Risk
A. I, IV
B. II
C. II, III
D. I, II, III, IV
A

B - At this time very little credit, marketability, and default risk exists.

How well did you know this?
1
Not at all
2
3
4
5
Perfectly
56
Q
At the beginning of the year, one U.S. dollar could buy 80 Japanese yen. At the end of the year, one U.S. dollar could buy 100 Japanese yen. What happened to the U.S. dollar during the year?
A. The U.S. dollar was revalued.
B. The U.S. dollar was devalued.
C. The U.S. dollar was inflated.
D. The U.S. dollar was deflated
A

A - By definition, the U.S. dollar was revalued. Revaluation refers to an increase in the currency’s value.

How well did you know this?
1
Not at all
2
3
4
5
Perfectly
57
Q
David invests $10,000 (U.S. dollars) in Tex Mex Foods (Mexican Exchange) when the exchange rate is 29 pesos to the dollar.  Tex Mex increases in value by 20%.  If David sells Tex Mex when the exchange rate is 30 pesos to the dollar, what will he receive in U.S. dollars?
A. $10,900
B. $11,600
C. $12,100
D. $12,414
A
B - Initially invested $10,000 x 29 =
290,000
Pesos
\+ 20% increase in Tex Mex value
58,000
Pesos
348,000
Pesos Convert to U.S. dollars 348,000 Ö 30 = $11,600
How well did you know this?
1
Not at all
2
3
4
5
Perfectly
58
Q
The prior question (# 14) is an example of which of the following?
A. Currency futures
B. Exchange rate risk
C. Devaluation
D. Revaluation
A

B - Nasty. This is an example of the uncertainty of returns after the investor converts back to the original currency. Depending on your perspective, both answers C and D are correct. Two things have occurred. The Mexican peso has been devalued versus the U.S. dollar, and the U.S. dollar has revalued versus the peso, so the answer must be exchange rate risk ? a forced answer.

How well did you know this?
1
Not at all
2
3
4
5
Perfectly
59
Q
If a fund has a beta of 1.05 in relation to the S&P 500, how much would the fund be expected to increase if the S&P 500 increased by 15%?
A. 14.25%
B. 15%
C. 15.75%
D. 22.5%
A

C - 15% x 1.05 = 15.75%

How well did you know this?
1
Not at all
2
3
4
5
Perfectly
60
Q
If a fund has a beta of 2.4 in relation to the S&P 500, how much would the fund be expected to move if the S&P 500 decreased by 10%?
A. Lose 2.4%
B. Lose 10010
C. Lose 24%
D. Lose 76%
E. Lose 14%
Investment Planning Quiz ? Lesson 4
A

C - 10% x 2.4 = 24%
The S&P decreased.
Investment Planning Quiz ? Lesson 5

How well did you know this?
1
Not at all
2
3
4
5
Perfectly
61
Q
Mrs. Bean lives in New York City (Manhattan). She is in a 28% federal tax bracket and pays 10% NY state tax and 5% NY city tax. If she purchases Treasury bonds that pay 10%, what is her after?tax rate of return?
A. 5.7%
B. 6.2%
C. 7.2%
D. 8.5%
E. 10%
A

C - Treasuries are not subject to state or city taxes but are subject to federal tax. 10% (1?.28) = 7.2%

How well did you know this?
1
Not at all
2
3
4
5
Perfectly
62
Q
Steven recently purchased a zero?coupon bond for $375. It will mature in 14 years, at which time it will be worth $1,000. What is the yield to maturity?
A. 7.01%
B. 7.13%
C. 7.26%
D. 7.42%
A

B - NOTE: Zeros are calculated using semiannual periods (in end mode).
HP 10B HP 12C HP 17BII
YTM 7.113% $375, ±, PV $375, CHS, PV $375, ±, PV
$1,000, FV $1,000, FV $1,000, FV
14, gold, xP/YR 14, enter, 2, x, n 14, gold, N I/YR i, 2, x I%YR

How well did you know this?
1
Not at all
2
3
4
5
Perfectly
63
Q
Sherry recently purchased a bond for $950 with a 6% coupon. It will mature in 20 years, but it can be called in 10? years at $1,050. What is the yield?to?call?
A. 6.87%
B. 6.69%
C. 7.06%
D. 7.28%
A

C - HP 10B HP 12C HP 17BII
YTC 7.06% $950, ±, PV $950, CHS, PV $950, ±, PV
$1,050, FV $1,050, FV $1,050, FV
10, gold, xP/YR 10, enter, 2, x, n 10, gold, N
60, Ö, 2, PMT 60, enter, 2, Ö, PMT 60, Ö, 2, PMT I/YR i, 2, x I%YR

How well did you know this?
1
Not at all
2
3
4
5
Perfectly
64
Q
Tom purchased a bond for $950 that has a coupon rate of 11%. The bond matures in 17 years and is callable in 5 years at $1,100. What is the YTC for this bond?
A. 11.69%
B. 12.89%
C. 13.87%
D. 14.02%
A
C - Use end mode/semiannual.  Solution: 13.87%
HP 10B
HP 12C
HP 17BII
Set for 2 P/YR
Set for 2 P/YR
5, gold, x P/YR
10, n
5, gold, N
$950, ±, PV
$950, CHS, PV
$950, ±, PV
$1,100 FV
$1,100 FV
$1,100 FV
$55, PMT I/YR
$55, PMT
$55, PMT
I/YR
i, 2, x
I%/YR
How well did you know this?
1
Not at all
2
3
4
5
Perfectly
65
Q
The annual returns for XYZ for the past three years were 8%, 12%, and ?6%. What is XYZ's geometric return?
A. 4.12%
B. 4.37%
C. 4.67%
D. 8.64%
A

B - (1.08 x 1.12 x .94) = (1.137)
1.137 FV, 1 ± PV, 3n = 4.37%
See Lesson 5 pages 1 and 2

How well did you know this?
1
Not at all
2
3
4
5
Perfectly
66
Q
A client purchased Steel, Inc., for $20,000. The stock paid a $1,000 dividend per year. The client sold the stock for $25,000 after two years. What is the holding period return?
A. 20%
B. 25%
C. 30%
D. 35%
A

D - HPR $25,000 + $2,000 ? $20,000 = 35%

$20,000

How well did you know this?
1
Not at all
2
3
4
5
Perfectly
67
Q
Alice is in a 31% bracket. She owns $10,000 of public purpose municipal bonds. They pay her $280 in interest semiannually. What pretax yield on corporate bonds is comparable to the yield on Alice's municipal bonds?
A. 2.8%
B. 4.06%
C. 5.6%
D. 8.12%
A

D - $28 x 2 = 5.6%
$1,000
5.6% = 8.12%
1 ? .31

How well did you know this?
1
Not at all
2
3
4
5
Perfectly
68
Q
Don Watson recently purchased a bond for $1,135 with a 10% coupon. It will mature in 10 years, but  can be called in 5 years at $1,100. He sells the bond in two years for $1,200. What is the holding period return?
A. 5.70%
B. 14.53%
C. 23.35%
D. 26.50%
A

C - [($1,200 + $200) ? $1,135] Ö $1,135 = 23.35%

How well did you know this?
1
Not at all
2
3
4
5
Perfectly
69
Q
Harry buys stock at $60 per share (100 shares on margin (50%). The margin interest is 12% annually. After 3 months, he sells the stock for $65. What is his holding period return?
A. 6.83%
B. 8.83%
C. 13.67%
D. 16.67%
A

C - ($6,500 ? $3,090) ? $3,000 = $410 = 13.67 (H PR)
$3,000 $3,000
Factor in the margin interest ($90 for the quarter). He only bought 100 shares. He only paid for 50 shares. The other 50 shares were bought on margin.

How well did you know this?
1
Not at all
2
3
4
5
Perfectly
70
Q
What is the current yield if a $1,000 bond with a 7% coupon is now selling for $1,050?
A. 6.23%
B. 6.67%
C. 7%
D. 7.33%
A

B - This is a premium bond. Current yield will be less than coupon rate.
Current Yield = $70 = 6.67%
$1,050

How well did you know this?
1
Not at all
2
3
4
5
Perfectly
71
Q
Todd buys $20,000 of stock on margin. He deposits $10,000 with the brokerage firm. The interest on the margin account is 12% compounded quarterly.  If he sells the stock exactly one year later for
$25,000, how much does he make?
A. 24.9%
B. 38%
C. $3,745
D. $3,900
A
C - 12% annually or $1,200 is answer B. $25,000 ? ($10,000 + $1,255) ? $10,000 = $3,745
10B
12C
17BII
Set for 4 P/YR
Set for 4 P/YR
$10,000 ± PV
$10,000 CHS PV
$10,000 ± PV
12i
12 Ö 4i
12i
1, gold, XP/YR
1 x 4n
1 gold N
FV = 11,255
FV
FV
FV
11,255 ? 10,000 = $1,255 interest
How well did you know this?
1
Not at all
2
3
4
5
Perfectly
72
Q
Bonds ...
I. pay interest at the beginning of the period.
II. pay interest at the end of the period.
III. are issued at par value
IV. pay semiannual interest
A. I, III
B. I, IV
C. II, III, IV
D. III, IV
Investment Planning Quiz ? Lesson 5
A

C - Investment Planning Quiz ? Lesson 6

How well did you know this?
1
Not at all
2
3
4
5
Perfectly
73
Q
Which of the following is not factored in calculating the intrinsic value using the DDM (Dividend Discount Model)?
A. Beta
B. Gross Earnings
C. Dividends paid
D. Risk?free return
A

B - Beta is used to calculate the required rate of return. r = rf + (rm ? rs). B that is used in Answer A and D. The DDM used the dividend paid (D0). Gross earnings (Answer B) is not a factor.

74
Q

Which of the following statements is true?
A. The shorter the time is to maturity, the greater the potential for a bond’s price fluctuation.
B. Duration is directly related to coupon rate.
C. Duration and maturity are positively correlated.
D. Duration is directly related to yield to maturity.
E. The higher the market interest rate is, the greater is the relative price fluctuations of bonds.

A

C - The other answers are incorrect.

75
Q

What is the approximate change in the price of a $1,000 bond when interest rates increase by 1.56%, the bond duration is 8 years, and the yield to maturity is 4%?
A. ?$113.68 B. ?$120.00 C. +$113.68 D. +$120.00

A

B - ?8 {.0156 + 1.04} = ?8(.015) = ?12%

$1,000 x ?12% = ?$120.00

76
Q
Which bond would be the least affected by an interest rate change?
A. Short duration bond
B. Long maturity bond
C. Low coupon bond
D. Large duration bond
A

A - The other bonds would be affected more.

77
Q
Which bond would be most affected by an interest rate change?
A. Short duration bond
B. High coupon bond
C. Short maturity bond
D. Long maturity bond
A

D - The other 3 bonds would be little affected. Notice, the question doesn’t ask what is least affected.

78
Q
If interest rates rise, which of the following would experience the greatest percentage drop in value?
A. T?bills
B. Treasury notes
C. Treasury bonds? 30 year
D. GNMAs
A

C - The longer the maturity, the more volatile the price. The longest maturity is T?bonds.

79
Q

A stock is selling for $28.50. A financial analyst determines that the proper P/E ratio is 50 and that the firm will earn $.50. The stock pays no dividend.
A. The stock is undervalued.
B. The stock is overvalued.

A

B - Current market price = $.50 x 50 =$25. At $28.50 per share, the stock is overvalued.

80
Q

You are given the following information on a publicly?traded company.
Book Value $10/share
Stock outstanding 10 million shares Common dividends paid $.20 share
EPS $1.90
What is the return on common equity from the data above?
A. 2.1%
B. 10.53%
C. 19%
D. 20.2%

A

C ROE = $1.90 = 19%

$10

81
Q

You are given the following information on a publicly?traded company.
Book Value $10/share
Stock outstanding 10 million shares Common dividends paid $.20 share
EPS $1.90
What is the dividend payout ratio from the data above?
A. 2.1%
B. 10.53%
C. 19%
D. 20.2%

A

B - Dividend Ratio = $.20 = 10.53%

$1.90

82
Q
XYZ Company's stock closed yesterday with the following data.
Closing Price
$50
Dividend
$2
Earnings per Share
$3
Trading Range
$50 ? $52
What is the stock's yield?
A. 4.0%
B. 6.0%
C. 66.7%
D. 5.8%
E. 3.8%
Investment Planning Quiz ? Lesson 6
A

A - $2 Ö $50 = 4%

Investment Planning Quiz ? Lesson 7

83
Q
A portfolio that is positioned outside (above) the efficient frontier is:
A. Unattainable
B. Attainable
C. Inefficient
D. Optimal
A

A Unattainable/not feasible

84
Q
An indifference curve of a highly risk adverse investor is:
A. Flat
B. Inverse
C. Steep
D. Exponential
A

C

85
Q
If Rm (the return on the market) is 10% and Rf (the risk?free rate) is 6%, then what is the stock risk premium if the security has a beta of 1.4?
A. 5.6%
B. 8.4%
C. 10%
D. 14%
A

A - (10% ? 6%) 1.4 = (.04) 1.4 = 5.6% (The formula for stock risk premium)

86
Q
Which form of Efficient Market Hypothesis reflects all information including insider information?
A. Strong Form
B. Semi?strong form
C. Weak form
D. Strong and semi?strong forms
E. All three forms
A

A - Strong form fully reflects all information, public or private.

87
Q

Which of the following are consistent with belief in the EMH?
I. An individual investor can randomly select a diversified portfolio of securities and earn a return consistent with the market as a whole.
II. Once a portfolio has been selected, there is no need to change it.
III. Technical analysis out performs a buy?and?hold strategy.
IV. The prices of securities do not reflect all available information.
A. I, II, III
B. I, II, IV
C. I, II
D. III, IV
E. I, IV

A

C - The prices of securities fully reflect all information. Technical analysis will not produce superior results.

88
Q
What is Mr. Pate's required rate of return if the Rf (T?bill rate) is 4.5%, his Beta is 1.0, and the Rm (market return) is 600 basis points above the T?bill rate?
A. 4.5%
B. 6%
C. 9%
D. 10.5%
A

D - r = Rf + (Rm ? Rf) B (The required rate of return) r = 4.5% + (10.5% ? 4.5%) 1 = 10.5%

89
Q
Modern portfolio theory has a risk?return relationship based on which of the following?
A. Weighted beta
B. Correlated coefficient
C. Alfa
D. Capital Market Line
E. Security Market Line
A

D - This is the best answer based on the following. The CML specifies the relationship between the risk and return on a variably?weighted market portfolio consisting of all risky assets. When you read the material on lesson 7 page 1, this sounds like the CML. However, it is a difficult question to answer.

90
Q
What is the most fundamental, distinguishing characteristic of the Markowitz efficient frontier?
A. Return
B. Covariance
C. Risk
D. Correlation
A

C - Answers A, B, and D play a supportingrole in determining the efficient frontier. But, answer C is the most fundamental, distinguishing characteristic of the efficient frontier.

91
Q

In regard to the CML, which of the following is true?
I. The CML can evaluate a diversified, margined portfolio.
II. It can be used to evaluate a security’s performance.
III. The CML includes a portfolio of 100% treasury bills.
IV. The point of tangencyof the CML with the efficient frontier represents a portfolio with a proportional percentage of all possible risky assets.
A. I, II
B. I, II, IV
C. I, III, IV
D. II, III
E. III, IV

A

C - The CML cannot be used to evaluate a single security.

92
Q

Which of the following statements concerning risk tolerance is (are) correct?
I. Emotions can severely limit a person’s ability to make rational decisions about risk.
II. A person with a low net worth who has a $5 million umbrella liability policy is highly tolerant in financial matters.
A. I
B. II
C. I, II
D. Neither

A

A - A person who has a low net worth and a large umbrella liability insurance policy is probably risk averse.

93
Q

Random walk hypothesis suggests which of the following.
A. That technical analysts can outperform the market.
B. That fundamental analysts can outperform the market.
C. That the next price change of a stock is unrelated to past price behavior.
D. That security pricing reflect all known information.
Investment Planning Quiz ? Lesson 7

A

C - If prices move randomly, technical analysis is useless. Weak form is related to, but not identical with, random walk; therefore, fundamental analysis could be an answer. Answer C is the definition of random walk, and Answer D is the definition of EMH.

94
Q
When dealing with uncertain variables, you define the possible values with a probability distribution. The types of distribution include which of the following?
I. Normal
II. Triangular
III. Uniform
IV. Lognormal
A. None of the above
B. I, III
C. I, II, IV
D. I, III, IV
E. All of the above
A

E - There are more than a dozen different probability distributions. You should know the four distributions listed in the Problem.

95
Q
Which index is price weighted?
A. DJIA
B. S&P 500
C. EAFE
D. Value Line
A

A

96
Q

Which of the following is true about Dow Theory?
A. The theory uses EMH principles.
B. The most important price movement is day?to?day fluctuations.
C. It is a method that identifies the top of a bull market and the bottom of a bear market.
D. Primary price movements are based on Modern Portfolio Theory.

A

C - Dow theory contradicts Modern Portfolio Theory and EMH. It is based on trends, not day to day fluctuations.

97
Q

Assume the returns on a managed portfolio are regressed against the returns on a market index. The resulting alpha shows which of the following?
A. The amount of the portfolio’s price movement explained by the market.
B. The degree of diversification in the portfolio.
C. The amount of unsystematic risk in the portfolio.
D. The return added to the portfolio by the portfolio manager

A

D - Alpha indicates how the portfolio manager performed.

98
Q

The Sharpe index does which of the following?
I. Assumes the portfolio is well?diversified.
II. Assumes the portfolio is not diversified.
III. Compares the actual return to the expected return.
IV. Standardizes performance by the portfolio’s Beta coefficient.
A. I, III, IV
B. II, III
C. II
D. I, II

A

C - Answer I, III, and IV refer to the Jensen index.

99
Q

Given the following data on Fund X.
Realized return of Fund X 17% Realized return of market 18% Beta Fund X .72 Beta of Market 1.0 Standard deviation Fund X 9% R2 Fund X 87%
If the risk?free rate is 6%, what is the alpha of the fund?
A. ?3.64%
B. 2.36%
C. 8.36
D. 13.16

A

B - ap = rp?[rf+(rm?rf)Bp] = 17% ? [6% + (18% ? 6%) .72] = 2.36
The question asked you to calculate the alpha. There is no shortcut to the calculation.

100
Q
A client owns 10,000 shares of a stock worth $2 a share. The company declares a 10 for 1 reverse split. After the split, how many shares will the client own?
A. 500
B. 1,000
C. 2,000
D. 20,000
A

B - Before the split, 10,000 at $2 = $20,000 After the split, 1,000 at $20 = $20,000

101
Q

Financial ratios are useful for analysis using which of the following?
A. A single ratio across different industries over time
B. A single ratio within the same industry over time
C. Several ratios across different industries over time
D. Several ratios with the same industry over time

A

D - One ratio by itself means little (Answer B), but several ratios together may give a clear picture of the firm’s strengths and weaknesses. Then they may be compared at a given time for several firms within the same industry.

102
Q
A client has the following assets and liabilities.
Checking account
$3,000
XYZ REIT
$20,000
Money market
$12,000
Credit Card Debt
$ 5,000
401(k) Balance
$50,000
Mortgage
$50,000
What is the current ratio?
A. .4
B. 1.55
C. 5
D. 7
A

D - checking/money market/REIT = $35,000 = 7 credit card debt $ 5,000

103
Q

Which portfolio is the riskier portfolio considering correlation?
I. 50% in the S&P 500 index and 50% in the Russell 2000 index
II. 50% in the S&P 500 index and 50% in the EAFE index
A. I
B. II

A

A - This is a correlation question. Answer II will have a lower correlation which will reduce the variance of the portfolio. The question is asking about the riskier portfolio. Subjective.

104
Q
An investor wants to immunize his/her bond portfolio. Immunization will lower which of the following?
A. Interest rate risk
B. Purchasing power risk
C. Principal risk
D. Liquidity risk
E. Market risk
A

A - Duration is used to compare interest rate risk between bonds with different maturities and coupons. It can be used to immunize a bond from Joss of principal due to interest rate changes.

105
Q

Which statement concerning straddles is correct?
A. It is simultaneously buying a call and writing a call on the same stock but with different exercise prices or exercise dates.
B. It is simultaneously buying a put and call on the same stock with the same exercise price and expiration date.

A

B - Answer A describes a spread.

106
Q
Active investment strategies include which of the following?
I. Dollar cost averaging
II. Mutual fund switching
III. Market timing
IV. Buying an Index fund
A. I, II, III
B. II, III
C. II, IV
D. IV
A

B - Dollar cost averaging and buying an Index fund are passive investment strategies.

107
Q
Your client buys 1,000 sl1ares of high?flying Internet stock at $60. The initial margin is 50%. You inform him that this stock has a 30% maintenance margin. Below what trade price do you tell him to expect both a telephone call from you and margin call?
A. $18
B. $30
C. $40
D. $44
E. $42.85
A

E - 1?.50 x $60 = .5 x 60 = $42.85

1?.30 .7

108
Q
With Regulation T at 50%, your client purchases 100 shares of XYZ at $70 per share and simultaneously purchases an XYZ May 70 put for a premium of $4. What is the minimum amount of cash the client must put up using margin?
A. $3,700
B. $3,900
C. $7,000
D. $7,400
A

B - The 100 shares of stock at $70 are marginable at 50%. Options are not marginable securities. The full amount of the option cost must be paid in cash. 50% of $7,000 plus the put premium of $400 is $3,900.

109
Q
Adam has been purchasing a mutual fund averaging approach. Adam has invested $10,000 each year for 5 years on a dollar?cost averaging approach.
Year
Investment
Share Price
1
$10,000
$120
2
$10,000
$100
3
$10,000
$118
4
$10,000
$97
5
$10,000
$130
If Adam sells 100 shares at $150 per share, what will be his gain using the average cost method?
A. $3,842
B. $5,000
C. $11,158
D. $15,000
A
A - $10,000
Ö
120
=
83.000
$10,000
Ö
100
=
100
$10,000
Ö
118
=
84.745
$10,000
Ö
97
=
103.092
$10,000
Ö
130
=
76.923
$50,000
448.093   = $111.58/share
Sells 100
@ $150
=
$15,000
Cost 100
@$111.58
=
?11.158
Gain
$3,842
110
Q

In the APT model, when a factor is zero, which of the following is true?
A. It’s expected or anticipated.
B. It’s unexpected or unanticipated.
C. The factors have an impact on the return.

A

A - When a factor is zero, the factor has no impact on the return because it is expected or anticipated.

111
Q

Which of the following is true about arbitrage pricing theory?
A. Security movements are explained by a relationship between risk and return.
B. The expected value of each factor is zero.
C. Pricing of securities in different markets can differ for significant lengths of time.
D. Unexpected changes in inflation and anticipated shifts in risk premium will influence security prices.

A

B - Answer D says anticipated shifts. This is incorrect for APT. APT says unanticipated shifts in risk premium will influence security prices.

112
Q

Under Black/Scholes, which of the following variables will decrease the value of a call option?
A. An increase in the price of the stock.
B. An increase in the strike price.
C. An increase in the time to expiration.
D. An increase in the volatility of the stock.
E. An increase in interest rates.

A

B - All other answers increase.

113
Q
Binomial option pricing is a	model
A. pricing
B. valuation
C. volatility
D. variability
A

B - Like Black?Scholes, it is a valuation model. Prices are established through the action of buyers and sellers; investors and analysts use valuation models to estimate what those prices should be.

114
Q
A risk tolerant client would not have the following characteristic.
A. Small amounts of insurance
B. Changes jobs
C. No debt
D. Experienced
A

C From page 10?1

115
Q
Which two investments would not have low correlations?
I. Equity investments
II. Preferred stocks
III. Real estate
IV. Bonds
A. I, II
B. I, III
C. II, III
D. II, IV
E. III, IV
A

D From page 10?2
Fixed income securities, like preferred stock and bonds, have low correlations with other asset classes shown. However, they are in the same asset class ? fixed income securities.

116
Q
Which of the following is not a passive asset allocation technique?
A. Advance/decline line
B. Laddered bonds
C. Dollar?cost?average
D. Indexed portfolios
A

A - The advance/decline line is a technical indicator. Technical indicators are active not passive.

117
Q
Which of the two aspects of the CAPM is used for an individual security?
A. CML
B. SML
C. Neither
D. Both
Investment Planning Quiz ? Lesson 10
A

B - The CML specifies the relationship between risk and return on a portfolio.
Investment Planning Final

118
Q
A person whose entire portfolio is comprised of bonds is most susceptible to which type of risk?
I. Interest rate risk
II. Reinvestment rate risk
III. Market risk
IV. Purchasing power risk
A. I, II, III
B. I, II, IV
C. I, III, IV
D. II, III, IV
A

B - Bonds are affected by DRIP. Although any security is subject to market risk, you were forced to pick the DRIP answer.

119
Q
Which of the following are liquid?
A. ETFs
B. CD (6 month term)
C. Government bond fund
D. Tech fund
E. Money market fund
A. I, II, III, IV
B. I, V
C. II, V
D. I, II, V
A

C - ETFs, government bond funds, and tech funds are not liquid.

120
Q
Which of the following is an unsystematic risk?
A. Management risk
B. Financial risk
C. Exchange rate risk
D. Purchasing power risk
A

B - Management risk might be a business risk. But from investments lesson 4, financial risk is the best answer.

121
Q

A client is interested in the PCT mutual fund. The beta is less than 1, the R2 is low, the standard deviation is high, and alpha is high. What should you tell the client?
A. Alpha is significant
B. Beta is significant
C. The Sharpe calculation is significant.
D. The Jensen calculation is significant.

A

C - Alpha, beta and Jensen are not significant because of a low R2. Sharpe, which is a function of the standard, deviation, is significant.

122
Q

A client calls you because she hears about a mutual fund. She asks you if the mutual fund is a wise investment and whether she should buy it. You research the mutual fund and find out the following. Alpha Beta R2 SD T?Bills
?7 .4 22% 18% 3%
What would you recommend?
A. Do not purchase the mutual fund because the alpha is negative 7.
B. Purchase the mutual fund because the beta is low.
C. Purchase the mutual fund because R2 is significant.
D. Do not purchase the mutual fund because the standard deviation is high.

A

D - With a low R2, the standard deviation (Sharpe) becomes important. The R2 is too low to use the alpha answer. Alpha measures (as a percentage) the contribution of the portfolio manager. Don’t buy a mutual fund just because its beta is low. With a standard deviation of 18%, this mutual fund will have a wider dispersion from the average return. Investors prefer assets with the least risk for a given expected return. Sometimes you are forced into an answer because nothing else makes sense. Answer B and C do not make sense. Many questions on the CFP Certification Exam are on the edge. You can consider this to be a similar question to what you might see.

123
Q

Which of the following statements concerning Beta is (are) correct?
I. It is a measure of a stock’s systematic risk.
II. A beta of 2 is 200% more volatile than an average stock.
III. Beta is an index of variability.
IV. Beta is a measure of total risk.
A. I, II, III
B. I, II
C. I
D. II, III, IV

A

C - A stock with a beta of 2 is 100% more volatile than an average stock.

124
Q
Which fund should you select and why?
Beta
R2
Treynor
Sharpe
Fund A
2.0
82
4.23
8.30
Fund B
1.9
80
3.0
10.91
Fund C
1.5
90
6.49
3.92
A. Fund A because it has the highest Treynor number
B. Fund C because it has the highest R2
C. Fund B because it has the highest Sharpe number
D. Fund B because it has the lowest Treynor number
E. Fund C because is has the highest Treynor number
A

E - When R2 is high, you would pick the highest Treynor number. Answer B is wrong. Performance isn’t based on R2.

125
Q
Input variables needed to create portfolios under the Markowitz model do not include which of the following?
A. Covariance
B. Correlation coefficient
C. Standard deviation
D. Return
E. Beta
A

E - Markowitz uses standard deviation as a measurement (covariance, correlation coefficient, and return).

126
Q
If point A represents the market, then which point(s) represent(s) overvalued investments?
I. A
II. B
III. C
IV. D
V. E
A. I, II
B. III
C. IV, V
A

B - Only point C is overvalued. Points D and E are undervalued. Please refer to page Lesson 7?10 of the textbook.

127
Q
The January effectand day of week effectaffect which form of Efficient Market Hypothesis?
A. Strong form
B. Semi?strong form
C. Weak form
D. Strong and semi?strong forms
E. None of the three forms
A

E - ‘January effect’ and ‘day of week effect’ (specifically Monday effect) are anomalies. A market anomaly is a strategy or situation that cannot be explained away but would not be expected to happen if the efficient market hypothesis were completely true.

128
Q
Which of the following forms of the efficient market hypotheses suggests that gain may be made by inside traders with access to illegal information?
A. Efficient Market Hypothesis
B. Semi?strong form
C. Strong form
D. Random walk
E. Weak form
A

B

129
Q

Sam is interested in buying XYZ at $25. As a financial planner, you have found the following information on XYZ and the market.
? Dividends are $1.40 currently and growing at 8%
? Beta of XYZ is 1.34. Standard Deviation of XYZ is 12.5
? Standard deviation of market is 14
? Yield on Treasury bills is 7.4%
? Market is expected to earn 12% What should you suggest Sam do?
NOTE: This uses two formulas.
1) The required rate of return formula. r = rf + (rm ? rf) B
2) The DDM formula V = D1 = D0 (1=q)
r?g r?g
A. Sam should buy the stock; it is overvalued.
B. Should buy the stock based on the data given.
C. The valuation of the stock is $27.17, and the stock should be purchased.
D. The valuation of the stock is $15.25, and the stock should not be purchased.
E. Not enough data is given to answer the question unless you calculate the required rate of return r = rf + (rm? rf).

A

C - Required rate (r) = .074+ (.12 ? .074)(1.34) = 13.564%
Intrinsic value = 1.40 (1 + .08) = 27.17 (buy at $25)
.13564 ? .08

130
Q
William recently purchased a zero?coupon bond for $400. It will mature in 12 years, at which time it will be worth $1,000. What is its yield to maturity?
A. 3.89%
B. 3.97%
C. 7.78%
D. 7.93%
A
C - Solve for YTM. Clear calculators.
10B
12C
17BII
$400, +, PV
$400, CHS, PV
$400, +, PV
$1,000, FV
$1,000, FV
$1,000, FV
12, Gold, xP/YR
12, enter, 2, x, n
12, gold, N
I/YR
i
I%YR
2, x
Answer: 7.78%
Note: Always assume semiannual compounding (even with zeros) unless told otherwise.
131
Q

When is a company most likely to issue new bonds?
I. When existing bonds are selling at a premium.
II. When existing bonds are selling at a discount.
III. When interest rates are expected to rise.
IV. When interest rates have fallen.
A. I, III
B. II, III
C. II, IV
D. I, III, IV
E. I, IV

A

D - A company would try to sell bonds before interest rates rise. In Answers I and IV, interest rates have fallen. The company could issue new bonds at the lower interest rates.

132
Q
Government bonds are not subject to which of the following risks?
A. Default
B. Reinvestment
C. Interest rate risk
D. Purchasing power risk
A

A Subject to RIP.

133
Q
If yield curve YC1 moves upward and becomes YC2, which of the following is true?
A. Duration has increased.
B. Duration has decreased.
C. Interest rates have decreased.
D. Only long?term bonds have increased.
A

B - If interest rates have increased, then duration has decreased (inverse relationships.)

134
Q

Which of the following is false?
A. The smaller a bond coupon, the more volatile the bond’s price.
B. The term and duration of a bond are equal for zero coupon bonds.
C. The smaller the duration, the more volatile the bond’s price.
D. Matching a bond’s duration with the time horizon reduces reinvestment risk.

A

C

135
Q

Which of the following is true?
A. The smaller the coupon, the greater the relative price fluctuation.
B. The longer the term to maturity, the less the relative price fluctuation.
C. The lower the market interest rate, the less the relative price fluctuation.

A

A

136
Q
What is the yield to maturity of an 8% coupon bond which can be purchased for $800 and will mature for $1,000 in 10 years?
A. 10%
B. 10.5%
C. 11.4%
D. 11.96%
E. 12%
A

C (End mode)

$800 CHS PV, $1,000 FV, 40 PMT, 20n. Solve for interest (i)

137
Q

Allan is a conservative investor with a financial goal of sending his son to college approximately 7 years from now. He wants to reduce the interest rate and reinvestment rate risk and is considering purchasing of one of the following three bonds:
Bond 1: AA rated, 7 year maturity, 5.60 duration, 8% coupon, selling for $995 Bond 2: AAA rated, 9 year maturity, 6.57 duration, 9% coupon, selling for $1,200 Bond 3: AA rated, zero coupon, 7 year maturity, selling for $500
Comparable debt currently is yielding 8.5%. Which bond should Allan purchase and why?
A. Bond 3 because it is selling for a greater discount than Bond 1 and Bond 2
B. Bond 1 because its maturity matches the goal time frame.
C. Bond 2 because its duration most closely matches the goal time frame.
D. Bond 3 because its duration matches the goal time frame.

A

D - The zero’s maturity equals is duration.

138
Q
Which entity normally purchases STRIPS?
A. The U.S. government
B. A U.S. corporation
C. A pension plan
D. A trustee for a revocable trust
A

C - STRIPS are normally purchased by pension plans because taxes are due on the interest, but no interest is paid until maturity (phantom income).

139
Q

Z Corporation had excess funds in a bank account (non?interest bearing). The funds will be needed in one year for capital improvements. What would you suggest?
A. Purchase a blue?chip common stock for capital gains.
B. Purchase a preferred stock (paying 4%) for the dividends exclusion.
C. Purchase a balanced mutual fund for dividends and capital gains.
D. Purchase a T?Bill for risk?free interest.
E. Purchase a T?Bond for risk?free interest.

A

D - The time element of one year makes all the other answers too risky or speculative. This again is a suitability question (time element).

140
Q

Mr. N is a nervous investor. He isn’t sure he should get into the bond market. He is uncertain about when he might need the investment money, possibly sooner than maturity. What would you suggest?
A. Buy bonds with a call feature.
B. Buy convertible bonds so he can convert them.
C. Buy bonds with a put feature.
D. Buy premium bonds.

A

C

141
Q
T?Bills are different from Treasury bonds in which ways?
I. Maturities
II. Risk
III. Callable
IV. Interest paid
V. Taxation
A. All of the above
B. I, II, III, IV
C. II, III
D. III, IV
A

B - Bills have short?term maturities; bonds have long?term maturities. Because of their maturities, they have different risks. Bills are not callable: T?bonds are callable. Bills are sold at a discount; bonds pay interest semiannually.

142
Q
In a time of declining interest rates, which of the following produces the most total return?
A. Corporate long?term bond
B. Zero bond
C. GNMA fund
D. T?bills
A

A - Only one bond indicates its maturity (Answer A). The zero does not indicate its maturity and produces only phantom income. When interest rates decline, mortgages are normally refinanced (GNMAs). T? bills only last for 1 year (or less). When interest rates decline, the client wants to own long bonds for capital gains.

143
Q
A client is considering buying bonds but is concerned about rising interest rates due to inflation. What should he look for in the bond indenture?
A. Bonds with a sinking fund
B. Bonds with a put feature
C. Bonds with a call feature
D. Bonds with a conversion feature
A

B - A put bond allows its holder to redeem the issue at specified intervals before maturity and to receive face value. Inflation will drive down the bond value. Inflation will also drive down stock values and make the conversion feature worthless. If you bought a bond paying 5% and now due to inflation bonds were paying 10%, you would like to redeem your 5% bond and buy a 10% bond, but the value of the 5% bond has gone down.

144
Q
Aunt Sally has various CD accounts with family members at the same bank.
$40,000 CD with her brother	(joint)
$40,000 CD with her sister	(joint)
$100,000 CD with her mother	(joint)
$20,000 CD with her niece	(joint)
$20,000 CD with her nephew	(joint) She is covered in what amount?
A. $100,000
B. $110,000
C. $200,000
D. $210,000
E. $220,000
Question 28
A

B - She is covered for an aggregate maximum of $110,000.

145
Q
continues. Other family members are covered (not including Sally) for what amount?
A. $80,000
B. $100,000
C. $110,000
D. $210,000
E. $220,000
A
C - $20,000 brother
$20,000 sister
$50,000 mother
$10,000 niece
$10,000 nephew
$110,000
Yes, there is a total of $220,000 of coverage, but that's not the question.
146
Q

Mrs. Pell, age 70, is concerned about the possibility of having to go into a nursing home. She has approximately $500,000 in various investments but most are in long?term quality investments. She has no LTC. What is a good way to allocate Mrs. Pell’s investments?
A. Liquidate all investments and invest in money market instruments.
B. Purchase $70,000 of laddered CDs (maximum maturity one year) and leave the remainder alone.
C. Purchase $500,000 of a combination of CDs, T?bills, and money market Instruments.
D. Purchase $70,000 of treasury bonds and leave remainder alone

A

B - The $70,000 might pay for a good part the first year of LTC. The question says possibility of going,Ó not that she will be going into a nursing home. That is a tough question without knowing the time frame, or where she lives. LA and NYC would be very expensive, but Tallahassee, FL, might be reasonable. This is a subjective question/answer.

147
Q

A lady feels that a clothing retailer has fallen out of favor. What would you advise her to do?
A. Buy a call for $300.
B. Buy a put for $300.
C. Write a put for $300.
D. Sell the stock short at $50 per share.

A

B - If she buys a put for $300, that’s the maximum she can lose. If she sells the stock short and the stock goes up, she has to buy the stock on the open market to close her position.

148
Q
What type of option offers the highest potential for loss?
A. Writing a covered put
B. Writing a covered call
C. Writing a naked put
D. Writing a naked call
E. Buying a call
A

D - A naked option loses $100 for every $1 increase in the price of stock above the call’s exercise price.

149
Q
A client buys an October 40 call at $3. The stock is currently selling at $39. What is the intrinsic value of the option?
A. ?1
B. 0
C. 3
D. 4
A

B - The option is out of the money.

150
Q
A client wanted to buy a stock at $50 but missed the opportunity. The stock is currently trading at $53. What could he/she do?
A. Buy 3 puts at $50
B. Sell 3 puts at $50
C. Buy 3 calls at $50
D. Sell 3 calls at $50
A

C - Answer B is also true, but the gain is limited to the put premium. The client feels the stock price will rise. He is optimistic. There is a lot less risk of buying a call (intrinsic value $3/share) than buying the stock at $53.

151
Q
Mrs. Cutter has a large capital gain in a stock. She is concerned that the stock price may decline and she will lose her gain. She does not want to sell the stock this year or possibly even next year for tax reasons. What would you advise her to do to defer her gain?
A. Sell a straddle
B. Sell a call option
C. Sell a put option
D. Sell the stock short
E. Buy a put option
A

E - Options have a maximum life of 9 months whereas selling short has no time element. Generally, you report a short sale in which you close the short sale by delivery of the replacement stock. However, this rule does not apply when you execute a short sale while holding the appreciated position in the same stock (short sale against the box). You report the transaction in the year of the short sale although delivery of the replacement stock is made in a later year. She does not want to sell the stock or be taxed this year or even next year. This is a covered position best answer is E.

152
Q

What are the differences between calls and warrants?
I. Call options are sold and bought by individuals; warrants are issued by corporations.
II. Call options have a limit of 9 months; warrants can have longer maturities than 9 months.
III. No new stock is created with a call option; warrants create new stock when exercised.
IV. Call option terms are not standardized; warrant terms are standardized.
A. All of the above
B. I, II, III
C. I, II
D. II, III, IV
E. I, IV

A

B

153
Q
What is not true about no?load funds?
A. They have no sales charges.
B. They have no management expenses.
C. There is a continuous offering and redemption of shares.
D. The shares sell at NAV.
E. The shares are redeemed at NAV.
A

B - All funds charge shareholders an expense fee for operating expenses (management fee).

154
Q
A client has the following mutual funds:
Fund
Income
Beta
Market Value
AAA
$6,000
$100,000
$100,000
BBB
$5,000
$100,000
$160,000
CCC
$9,000
$100,000
$99,000
DDD
$7,600
$100,000
$130,000
What is the current yield and gross profit percentage for the portfolio?
A. 5.64% / 18.2%
B. 5.64% / 29.15%
C. 6.9% / 18.2%
D. 6.9% / 29.15%
A

B - Total income $27,600 (from income column)
Current Yield = $27,600 = .0564 = 5.64%
$489,000
Income $27,600 plus growth $89,000 = $116,600 Gross Profit = $116,600 = .2915 = 29.15%
The $489, 000 is the last column. The $400,000 is the first column. The difference is the gain for the year.

155
Q

Tom is reviewing various investments. He will need $20,000 in 2?4 years for a month?long vacation. The XYZ Mortgage REIT (non?publicly traded) looks attractive to him due to high interest rates. The income payout has been consistent at around 10%. Should he purchase a REIT?
A. No, non?publicly traded REITs w likely not be liquidin 2?4 years.
B. No, REITs are not marketable.
C. Yes, a mortgage REIT will be an inflation hedge and very marketable.
D. Yes, a mortgage REIT will be an inflation hedge and very liquid.

A

A - Publicly?traded REITs (Equity, Mortgage, and construction) are marketable. Mortgage REITs are not liquid. Nonpublic REITs and Real Estate limited partnerships are illiquid and may not be marketable. Liquidity is the ability to turn assets into cash without significant loss of principal. A mortgage REIT would not do well during inflationary times.

156
Q
A 25?year?old woman wants moderate to moderate?high growth for herIRA. Which of the following should she purchase?
A. An annuity
B. Growth stock
C. Growth mutual funds
D. Municipal bond fund
A

C - Not enough information is given about the annuity. Purchase of an individual stock would not give her moderate high growth, but a growth mutual fund (by definition) provides moderate high growth.

157
Q
Which investment could provide both leverage and a hedge against inflation?
A. Equity mutual fund
B. Equity REIT
C. Mortgage REIT
D. Blind pool
A

B - Equity REITs can be leveraged. Equity REITs own the properties providing the investor with a hedge against inflation. An equity mutual fund or a mortgage REIT is a poor hedge against inflation. In a truly inflationary time (1981), stock did poorly. Inflation hit 21%. Inflation is the biggest factor in this question. There is not enough information about the blind pool to use it as an answer.

158
Q
Tim buys a stock XYZ for $100 per share (100 shares) in January. During the year, it pays $2 per share in dividends. At year?end, the stock is worth $110 per share. What is his holding period return?
A. Zero (not sold)
B. 2%
C. 12%
D. 24%
A

C - HPR = $110 + 2 ? 100 = $12 = 12%
$100 $100
NOTE: Although Tim didn’t sell the shares, I believe you could still calculate the HPR.

159
Q
A client would use Index funds if he/she believes in which of the following?
A. Technical analysis
B. Fundamental analysis
C. Passive investment strategy
D. Active investment strategy
A

C - Investing in a mutual fund that replicates a market index, such as the S&P Index, thus assuring investment performance no worse (or better) than the market as a whole (passive).

160
Q
Stock
Stock Price Earnings  Dividend Yield Company A  $50   $2.00   4%
Company B $40   $2.00   5%
Company C $20   $2.00   7%
Company D $30   $2.00  6%
Which company is most likely to pay a dividend next year?
A. Company A
B. Company B
C. Company C
D. Company D
A

C - $20 x 7% = $1.40 dividend. The other companies’ dividends equal or are close to equaling the company earnings.

161
Q
What is the best way to buy a foreign stock?
A. Global fund
B. International fund
C. Buying overseas
D. ADR
A

D - The best way to buy an individual stock is to buy an ADR.

162
Q

John and Jody want to diversify internationally. Which alternative is the least attractive to them?
A. Investing in their existing mutual fund.
B. Investing in an open?end international mutual fund.
C. Investing in a closed?end international fund.
D. Investing in an open?end global mutual fund.
E. Investing in a closed?end global fund.

A

A - The question asked for the least attractive (adding more to existing fund).

163
Q

Tiffini, single and age 26, has an excellent job paying her in excess of $80,000. She has an adequate emergency fund. As a CFP practitioner, what would you suggest that she do with her excess savings?
A. Add to her money market account because she is single (emergency fund)
B. Buy municipal bonds because of her tax bracket.
C. Invest in a global growth fund to get diversification.
D. Invest in a domestic growth fund for diversification.
E. Invest in an individual stock (12%) to get more growth and less taxation.

A

C - Answer B is a good option (low bracket tax), but considering the facts. She could be in a 25% or lower marginal tax bracket. Even 25% is low for municipal bonds to be effective. This is covered in General Principles Lesson 5. Answer C is the best answer.

164
Q
A Canadian company which exports its products to the U.S. wishes to protect itself during a time in which the American dollar is expected to be devalued. The company should do which of the following?
I. Buy U.S. dollars
II. Sell U.S dollars
III. Buy Canadian currency
IV. Sell Canadian currency
A. I, III
B. III, IV
C. II, III
D. II, IV
A

C - If the company expects the U.S. dollar to be devalued, that means the Canadian currency will be worth more.

165
Q

Who among the following is considered an insider?
NOTE: This comes from General Principles ? Think Simple!
I. Employee of the firm
II. Officer
III. Director
IV. A person who has information about the firm that no one else has
A. All of the above
B. I, II, III
C. II, III
D. II, III, IV

A

D - I don’t think an employee would be considered an insider unless the employee had access to key information.

166
Q

John and Mary are considering buying some more Beta Corporation stock. What do you recommend?
A. Buy based on the PE ratio because the current price of the stock is overvalued
B. Buy based on the DOM because the current price of the stock is overvalued
C. Buy based on the PE ratio because the current price of the stock is undervalued
D. Do not buy based on the DOM because the current price of the stock is undervalued
E. Do not buy based on the DDM because the current price of the stock is overvalued

A

E - Based on the constant?dividend?growth formula (DDM), the current price of the stock is overvalued. Do not buy.
IV = D0( 1 +g) = $3(1 + .05) = 3.15 = $45 r Ðg .12 ? .05 .07
Using the EPS and the P/E, the stock is worth $60. Because of the high dividend and dividend growth information, CFP Board probably would want you to use the intrinsic value method. The P/E doesn’t indicate either undervalued or overvalued. EPS x P/E = $4 x 15 = $60.

167
Q
What was the yield to call of the local municipal bonds when John and Mary originally bought them if they are called?
A. 8.07%
B. 6.69%
C. 10.12%
D. 4.033%
E. 3.347%
A
A End Mode
10B (2 P/YR)
12C
17BII (2 P/YR)
950 ± PV
950 CHS PV
950 ± PV
1,050 FV
1,050 FV
1,050 FV
30 PMT
30 PMT
30 PMT
5 gold xP/YR
10n
5 gold N
4.033% x 2
8.07
8.07
8.07
168
Q
What is the YTM (Yield to Maturity) of the local municipal bonds when John and Mary bought them if they mature?
A. 8.07%
B. 6.69%
C. 10.12%
D. 4.033%
E. 3.347%
A
B - 10B (2 P/YR)
12C
950 ± PV
1,000 FV
30 PMT
10 gold xP/YR
6.69
950 CHS PV
1,000 FV
30 PMT
20n 3.347% x 2
6.69
End Mode
10B (2 P/YR)
12C
17BII (2 P/YR)
950 ± PV
1,000 FV
30 PMT
10 gold xP/YR
6.69
950 CHS PV
1,000 FV
30 PMT
20n 4.033% x 2
6.69
950 ± PV
1,000 FV
30 PMT
10 gold N
6.69
169
Q
What is the current tax?equivalent yield on the local municipal bonds?
A. 6.12%
B. 9.38%
C. 9.56%
D. 10.46%
E. 12.60%
A

C - .36 is the marginal tax rate given in the case material.
$60* = 6.12% 6.12% = 9.56%
$980** 1? .36
* The coupon rate is 6% of $1,000 = $60
** The bonds are currently worth $4,900 Ö 5 = $980 The $4,900 is on the financial statement.

170
Q
What is the YTM (Yield to Maturity) of the zero coupon bonds when they were purchased?
A. 3.5265%
B. 7.05%
C. 7.177%
D. 14.11%
A

B - For 10B End Mode ? 2P/YR, use 10 gold xP/YR and do not multiply by 2.
These are semi?annual keystrokes, which is why you have to multiply by 2. They work in all calculators 1P/YR.
For 12C $500 CHS PV, $1,000 FV, 20n = 3.5265% 3.5265% x 2 = 7.05%

171
Q
According to the capital asset pricing model (SML), what is the required rate of return of the Zebra Corporation shares?
A. 11.2%
B. 12.2%
C. 14.2%
D. 15.2%
A

C - r = Rf + (Rm ? Rf)B = .04 + (.10 ? .04)1.7 = .04 + .102 = .142 = 14.2%. This formula is known as the CAPM and SML, In Question 55, we used the client’s required rate of return given (12%). In this question, we calculate it for this particular stock using the return on the market of 10%. The formula gives us the answer.

172
Q

How does Zebra’s required rate of return (Question 54) compare to John’s required rate of return in the case data?
A. Not enough information is given to answer the question.
B. The required rate of return is higher than Zebra’s rate of return.
C. The required rate of return is lower than Zebra’s rate of return.
D. This is a risky security with a beta of 1.7. The required rate of return should not be compared to his expected rate of return.

A

C - Zebra’s return (14.2%) is higher than his 12% required return. The 14.2% is the answer to question 61.

173
Q

What is the range of one standard deviation of the blue chip mutual fund?
A. +2.23 to +24.83 B. ?20.37 to +20.37 C. ?20.37 to +24.83 D. ?2.23 to +24.83

A

C - ?20.37 to +24.83

2.23 + 22.60 = +24.83/2.23 ? 22.60 + ?20.37

174
Q
Of the four mutual funds, which one has the highest usable alpha for analysis purposes?
A. Blue Chip
B. Emerging growth
C. Keogh #1
D. Keogh #2
A

D - The emerging growth fund has too low a R2 for alpha to be usable. Keogh Fund #2 (R2 64) has the highest usage alpha of the three remaining funds. It is not the highest R2, it is the highest alpha.

175
Q
Of the four mutual funds, which one has the highest usable Sharpe ratio?
A. Blue Chip
B. Emerging growth
C. Keogh #1
D. Keogh #2
A

B - The emerging growth fund has the highest Sharpe ratio.

176
Q

Does the portfolio mix fit John and Mary?
A. The portfolio needs more blue chip stocks paying dividends.
B. The portfolio should have a small cap fund to provide more diversification.
C. The portfolio fits a young high?earner aggressive investor.
D. The portfolio needs more zeros.
E. Not enough information is given to make a decision.

A

C - The Keogh portfolio on page 12 fits a young high?earner aggressive investor. Only his orientation is described (growth). This is my answer. You may disagree with Answer C. Blue chip stocks paying dividends sounds rather plain to a young high?earner individual. The small cap answer is good, but maybe it should have been more specific like 10% into a small cap. The zero would produce phantom income. I don’t think it fits this case. Subjective ? you can disagree ? still subjective.

177
Q

Bob, a CFP practitioner, suggests to his clients that they invest using dollar cost averaging. Why is he suggesting this method?
A. To take advantage of rising markets
B. To increase investment returns
C. To outperform lump?sum strategies
D. To teach his clients to be disciplined investors

A

D - All these answers are good. The markets may rise. It may increase investment returns, and it may
outperform Iump?sum.

178
Q
What is the difference between T?bills and T?notes?
I. Risk
II. Type of buyer
III. Denominations
IV. Cash flow
A. I, II
B. I, II, III, IV
C. II, III, IV
D. III
A

B - T?bills are considered the safest of all possible assets (riskless). The primary buyers of bills are corporations with excess short?term cash or money market funds. Individual investors usually purchase bills indirectly by buying money market accounts. Denominations are different (T?notes maximum of
$100,000). T?bills are sold at discount and mature in 12 months or less. T?bills do not pay interest and they mature for full face value. T?notes pay interest every 6 months.

179
Q
Mr. Thomas is going to start a 529 plan for his grandson, age 7. Which investment choice would you recommend?
I. Growth fund
II. S&P index fund
III. GNMA fund
IV. Emerging markets fund
V. Target?date fund
A

E - This is a tough, subjective answer. Growth and S&P index funds are too similar. It is hard to say that one is better. That’s why I eliminated both. If you answered growth or S&P, I am not saying you are wrong. Emerging markets seems too aggressive. GNMA might also be a good answer. But this can trigger a kiddie tax. Any additional bit of information can swing this answer. A target?dated fund seems the best answer. It resets to the time frame.

180
Q

Mrs. Jackson, age 65, just inherited a $500,000 IRA from her deceased husband. She states that she needs approximately $50,000 a year to maintain her lifestyle. In addition, her Social Security benefits will increase to $1,200 per month. How do you recommend she invest the $500,000 IRA if she considers herself conservative?
I. A balanced portfolio of stocks and bonds
II. A growth mutual fund
III. A FDIC insured account earning 6% {5 year CD)
IV. A 30?year treasury bond paying 7% (current yield)

A

D - She is conservative; therefore, answer D is better than answer A or B. It says approximately. The bond generates $35,000 of income per year without invading principle. Considering that plus the $14,400 from SS is approximately $50,000 (close enough). She needs the income now. She probably has a 25?30 year life expectancy. Inflation is NOT an issue in the question. Answer C will have potentially significant reinvestment risk at maturity (5 years) and falls too short of her income objective. The FDIC account would only be insured for $250,000 (under current law).
Can she take RMDs? Yes, she may have to take them. $500,000 divided by 27.4 is only $18,246. But, she is going to withdraw $35,000. There should be no RMD problem for many years.

181
Q
An American investor bought 10,000 shares of a Japanese stock when the yen was trading at 120 yen to the dollar. After purchasing the stock, the stock increased in value by 15%. The investor then sold the stock when the yen was 130 to a dollar. What was the investor's return?
A.   No solution B.   ?3.84%
C.   +6.15%
D.   +8.45%
Investment Planning ? Final
A
C Keep this simple:
$1 =	120 yen 1 + return		x1.15
138 yen
130 yen	= $1
130 yen	Ö 130   =	$1.0615
$1.0615	? $1   = .0615	= 6.15%
Income Tax Planning Quiz - Lesson 1